How to Solve a PDE using Laplace Transform?

Click For Summary
The discussion focuses on solving a partial differential equation (PDE) using the Laplace Transform. The initial transformation leads to a new equation involving U(s), where the integration factor is identified. The user struggles with finding the function A(s) and expresses concern about boundary conditions leading to undefined values. Eventually, the solution involves using partial fractions to simplify the expression, allowing for the successful resolution of the problem. The thread concludes with the user confirming that they have solved the PDE.
freesnow
Messages
10
Reaction score
0

Homework Statement



Use the Laplace Transform to solve the PDE for u(x,t) with x>0 and t>0:
x(du/dx) + du/dt = xt
with IC: u(x,0) = 0 and BC: u(0,t) = 0

Homework Equations



The Attempt at a Solution



After taking LT of the PDE wrt t, the PDE becomes
x(dU/dx) + sU = x/(s2)

Integrating factor :
I = exp(\int(s/x)dx) = xs

ODE becomes
d/dx(Uxs) = xs/s2

Integrating both sides:
U = x/(s3+s) + A(s)/xs

then I don't know how to find A(s), if I use BC, the factor 1/0 will come out...or is there some other way to calculate the PDE with LT?

thanks
 
Physics news on Phys.org
freesnow said:
Integrating both sides:
U = x/(s3+s) + A(s)/xs

then I don't know how to find A(s), if I use BC, the factor 1/0 will come out...

Unless A(s)=0 :wink:
 
Thanks!
 
so U(x,s) = x/(s3+s2)

but then I don't know how to do the inverse LT to get u(x,t) such that it fits the PDE...

thanks
 
freesnow said:
so U(x,s) = x/(s3+s2)

but then I don't know how to do the inverse LT to get u(x,t) such that it fits the PDE...

thanks

Just use partial fractions:

\frac{1}{s^3+s}=\frac{1}{s(s^2+1)}=\frac{A}{s}+\frac{Bs}{s^2+1}+\frac{C}{s^2+1}

Solve for A,B and C
 
I finally solved it! Thanks very much!
 
Question: A clock's minute hand has length 4 and its hour hand has length 3. What is the distance between the tips at the moment when it is increasing most rapidly?(Putnam Exam Question) Answer: Making assumption that both the hands moves at constant angular velocities, the answer is ## \sqrt{7} .## But don't you think this assumption is somewhat doubtful and wrong?

Similar threads

  • · Replies 11 ·
Replies
11
Views
2K
  • · Replies 5 ·
Replies
5
Views
2K
  • · Replies 10 ·
Replies
10
Views
2K
  • · Replies 1 ·
Replies
1
Views
1K
  • · Replies 11 ·
Replies
11
Views
2K
Replies
2
Views
2K
  • · Replies 3 ·
Replies
3
Views
3K
  • · Replies 7 ·
Replies
7
Views
2K
  • · Replies 1 ·
Replies
1
Views
971
Replies
9
Views
2K